You are on page 1of 59

Dear Aspirants

Team iLearn IAS is proud to present our weekly current affairs magazine.
Keeping in tune with the dynamic demands of the civil service examination, we
bring to aspirants on-going topics that are of relevance, particularly for the
mains exam.

The objective of this magazine is to make the aspirants exam ready in an


exhaustive list of topics. To this end, we provide detailed background notes on
the topics followed by an in-depth analysis of the issues, covering aspects such
as institutional mechanisms, policies, policy gaps, areas of focus, reforms
needed, suggestions and way forward.

We have also included all relevant Prelims topics from the past week.

In order to help the aspirants in monitoring their preparation, we have


included probable question that can be expected for the mains and Prelims
examination. You may also visit our integrated current affairs platform
www.ilearncana.com for your current affairs preparation.

We wish you all the very best and welcome you on board in our endeavors.

Happy Learning!
Team iLearn
TABLE OF CONTENTS
1. NUCLEAR WEAPON PROGRAMME OF INDIA .................................................................. 4
1.1 WHY IN NEWS? ................................................................................................................ 4
1.2 SYLLABUS: ........................................................................................................................ 4
1.3 PRACTICE QUESTION:....................................................................................................... 4
1.4 ANALYSIS: ......................................................................................................................... 4
1.5. iTips: .............................................................................................................................. 12
2. GENE EDITING AND CRISPR Cas9 ................................................................................ 13
2.1 WHY IN NEWS? .............................................................................................................. 13
2.2 SYLLABUS: ...................................................................................................................... 13
2.3 PRACTICE QUESTION:..................................................................................................... 13
2.4 ANALYSIS: ....................................................................................................................... 13
2.5. iTips: .............................................................................................................................. 17
3. LANDMARK JUDGMENTS PASSED BY THE SUPREME COURT IN 2019 ............................ 18
3.1 WHY IN NEWS? .............................................................................................................. 18
3.2 SYLLABUS: ...................................................................................................................... 18
3.3 PRACTICE QUESTION:..................................................................................................... 18
3.4 ANALYSIS: ....................................................................................................................... 18
3.5. iTips: .............................................................................................................................. 21
4.NATIONAL COMMISSION FOR PROTECTION OF CHILD RIGHTS ...................................... 22
4.1 WHY IN NEWS? .............................................................................................................. 22
4.2 SYLLABUS: ...................................................................................................................... 22
4.3 PRACTICE QUESTION:..................................................................................................... 22
4.4 ANALYSIS: ....................................................................................................................... 22
4.5. iTips: .............................................................................................................................. 25
5. GORKHALAND ISSUE ................................................................................................... 26
5.1 WHY IN NEWS? .............................................................................................................. 26
5.2 SYLLABUS: ...................................................................................................................... 26
5.3 PRACTICE QUESTION:..................................................................................................... 26
5.4 ANALYSIS: ....................................................................................................................... 26
5.5. iTips: .............................................................................................................................. 29
WEEKLY PRELIMS COMPILATION..................................................................................... 30
• Nitrous Oxide ........................................................................................................... 31
• Red Sea..................................................................................................................... 31
• GST compensation ................................................................................................... 33
• Rashtriya Kamdhenu Aayog ..................................................................................... 35

2 | iLearn IAS
• The Human Cost of Disasters report ........................................................................ 36
• Mega Food Park Scheme ......................................................................................... 37
• Tiger ......................................................................................................................... 38
• Anti-defection law.................................................................................................... 39
• Tuberculosis (TB) ...................................................................................................... 41
• STARS project ........................................................................................................... 43
• Pulses Cultivation in India ........................................................................................ 45
• Indian Rhinoceros .................................................................................................... 46
• Gupkar Declaration .................................................................................................. 47
• Clouded Leopard ...................................................................................................... 48
• Real Estate Appellate Tribunal (REAT) ..................................................................... 49
• National Health Authority ........................................................................................ 50
• Asia-Pacific Economic Cooperation ......................................................................... 51
• Global Hunger Index 2020 ....................................................................................... 52
• Caspian Sea .............................................................................................................. 54
• Thailand.................................................................................................................... 55
• Zoji La ....................................................................................................................... 56
• Integrated Farming .................................................................................................. 57

3 | iLearn IAS
1. NUCLEAR WEAPON PROGRAMME OF INDIA
1.1 WHY IN NEWS?
The UN General Assembly commemorates September 26, 2020 as the International
Day for the Total Elimination of Nuclear Weapons. In the meeting India reiterated that
nuclear weapons should be abolished in a step-by-step non-discriminatory process.
1.2 SYLLABUS:
GS 2: Bilateral, Regional and Global Groupings and Agreements involving India and/or
affecting India’s interests.
GS 3 : Various Security Forces and Agencies and their Mandate.
1.3 PRACTICE QUESTION:
Q. “Post-Cold War trend of gradual marginalisation of nuclear weapons may be reversed in
the current period of increasing protectionism and global mistrust”. Critically analyse
1.4 ANALYSIS:
BACKGROUND:
• The use of nuclear weapons in particular as well as other weapons of mass
destruction constitutes the gravest threat to humanity and to peace and stability in
the international system.
• Biological and chemical weapons have been outlawed by international treaties,
however nuclear weapons remain instruments for national and collective security,
the possession of which on a selective basis has been sought to be legitimised
through permanent extension of the Nuclear Non-proliferation Treaty (NPT) in May
1995
• Nuclear weapon states have asserted that they will continue to rely on nuclear
weapons with some of them adopting policies to use them even in a non-nuclear
context.
• These developments amount to virtual abandonment of nuclear disarmament. This
is a serious setback to the struggle of the international community to abolish
weapons of mass destruction
• India possesses both nuclear weapons and extensive nuclear fuel cycle capabilities.
• India tested its first nuclear device in May 1974, and remains outside both the
Treaty on the Non-Proliferation of Nuclear Weapons (NPT) and the Comprehensive
Nuclear Test Ban Treaty (CTBT).
• However, India has a facility-specific safeguards agreement in place with the
International Atomic Energy Agency (IAEA) and a waiver from the Nuclear Suppliers
Group (NSG) allowing it to participate in global civilian nuclear technology
commerce.
• India has a sizable and growing nuclear arsenal, primarily due to decades of conflict
with its nuclear-armed neighbor Pakistan

4 | iLearn IAS
HISTORY OF INDIA’S NUCLEAR WEAPON PROGRAMME:
• Developing a Peaceful Nuclear Program: 1947 to 1974:
o India's nuclear program was mainly conceived by Homi Bhabha, an
influential scientist who persuaded political leaders to invest resources in the
nuclear sector
o Government launched an ambitious nuclear program to boost the country’s
prestige and self-reliance in energy with primary focus on producing
inexpensive electricity.
o However, the decision to develop the complete nuclear fuel cycle also gave
India the technical capability to pursue nuclear weapons.
o In the years that followed, the internal debate over whether India should
develop a nuclear explosive device continued
o In the late 1960s nuclear scientists continued to develop the technical
capacity for a nuclear explosion
o Ultimately in 1974 India tested a fission device which it described as a
“peaceful nuclear explosion” (PNE)

• The slow path towards Weaponization: 1974 to 1998


o India’s 1974 nuclear test was condemned by many countries as a violation of
the peaceful-use agreements underlying U.S. and Canadian-supplied nuclear
technology and material transfers, and was a major contributing factor to
the formation of the Nuclear Suppliers Group (NSG).
o India authorized weaponization of India's nuclear capability in late 1980s as a
response to oblique nuclear threats issued by Pakistan in the wake of the
1986 to 1987 Brasstacks crisis
o At the same time, India continued to support efforts for nuclear disarmament
o Government authorized two rounds of nuclear tests in 1998, after which
India formally declared itself to be a nuclear-weapon state

• India as an Emerging Nuclear Power: 1998 to 2009


o India’s nuclear tests were followed within a month by a similar set of tests
by Pakistan, resulting in fears in the international community of an arms race
or an escalation of conflict
o The 1999 Kargil War and the 2001 to 2002 Twin Peaks Crisis heightened
tensions between the two countries, although these conventional conflicts
did not escalate to the nuclear level
o The US government imposed sanctions on both India and Pakistan in
response to their 1998 nuclear tests
o After the 1998 tests the Indian government established a National Security
Advisory Board, which issued a Draft Report on Indian Nuclear Doctrine in
1999

5 | iLearn IAS
o The country’s nuclear weapons remain under the control of the civilian
Nuclear Command Authority (NCA), comprising of a Political Council, chaired
by the Prime Minister, and an Executive Council, led by the National Security
Advisor.
o U.S.-India nuclear cooperation agreement unveiled in July 2005 and the
subsequent endorsement of India's case by the Nuclear Suppliers Group
(NSG), enabled India to engage in international nuclear trade.
o In return, India agreed to allow safeguards on a select number of its nuclear
facilities that are classified as "civilian" in purpose.
o The remaining "military" facilities remained off-limits to international
inspectors.
o U.S passed the Hyde Act in 2006 to exempt nuclear cooperation with India
from provisions of the U.S. Atomic Energy Act, allowing for the adoption of a
bilateral 123 nuclear cooperation agreement in August 2007.
o In 2008, the NSG approved an exemption allowing the members of this
export control regime to conduct nuclear trade with India.
o Finally, a safeguards agreement for select civilian nuclear facilities was
concluded between India and the International Atomic Energy Agency (IAEA)
in 2009.

• India as an Established Nuclear Power: 2009 to Present


o In 2009, India submitted a separation plan to put its civilian nuclear
facilities under IAEA safeguards by 2014
o In 2010, India and the United States signed a bilateral agreement allowing
India to reprocess U.S.-obligated nuclear material at two new reprocessing
facilities, to be constructed and placed under IAEA safeguards
o However, the nuclear power industry did not grow as expected because
India's liability laws regulating civilian nuclear power plants far exceeded
the international standards for nuclear liability and held suppliers legally
liable for any damages resulting from accidents.
o To address these concerns and give impetus to nuclear power industry, India
ratified the IAEA Convention on Supplementary Compensation for Nuclear
Damage in 2016
o By 2019, India had put total of 26 reactors under IAEA safeguards.
o Enabled by the NSG waiver granted to it in 2008, India has signed nuclear
cooperation agreements with Russia, United States, France, United Kingdom,
South Korea, Canada, Argentina, Australia, Sri Lanka, Japan, Vietnam,
Bangladesh etc.
o Additionally, India continues to participate in international nuclear trade and
has signed agreements with Canada, Kazakhstan and Australia to supply
uranium to fuel its civilian nuclear reactors.

6 | iLearn IAS
o Negotiations are currently underway for concluding negotiations to construct
six reactors in the Andhra Pradesh by Westinghouse.

• Recent Developments and Current Status:


o India was recently accepted as a member of three of the four major export
control regimes.
o It was admitted as a member into the Missile Technology Control Regime
(MTCR) in 2016, Wassenaar Arrangement in 2017 and Australia Group in
2018
o India has been actively pursuing membership into the NSG and has received
explicit support for its membership from many current NSG members
including the United States, Russia, Switzerland and Japan
o China’s opposition:
▪ However, China does not support an explicit membership in the NSG
for India but instead proposes a two-step approach: first would be to
reach consensus on a non-discriminatory resolution that would apply
to all non-NPT countries alike and then discuss individual membership
applications by non-NPT countries.
o India’s argument for NSG membership:
▪ In arguing for NSG membership, India has portrayed itself as a
responsible nuclear power, pointing to its positive record on non-
proliferation and consistent support for complete nuclear
disarmament.
▪ India argues that its membership should be considered under current
rules because NSG is an export-control mechanism and not a non-
proliferation one so question of linking NSG membership to the NPT
membership does not arise
▪ Furthermore, India argues that there is a precedent for non-
signatories of NPT joining the NSG when France became a founding
member of the NSG in 1974 but did not accede to the NPT until 1992
o India’s declared nuclear posture is of credible minimum deterrence and has
successfully developed a strategic triad of nuclear delivery systems.
o India has not signed the CTBT, but maintains a unilateral moratorium on
nuclear testing and supports negotiations for a Fissile Material Cut-off Treaty
(FMCT) that is "universal, non-discriminatory, and internationally verifiable."
o At the same time, India has remained firmly outside of the NPT, arguing that
“nuclear weapons are an integral part of our national security and will remain
so pending the global elimination of all nuclear weapons.”
o India maintains its official commitment to no-first-use of nuclear weapons.
However, during prepared remarks at the Pokhran nuclear test site in August

7 | iLearn IAS
2019, Indian Defence Minister implied that India’s no first use policy would
not be continued indefinitely

FEATURES OF INDIAN NUCLEAR DOCTRINE:


• Fundamental purpose of Indian nuclear weapons:
o The fundamental purpose of Indian nuclear weapons is to deter the use and
threat of use of nuclear weapons by any State or entity against India and its
forces.
• No First Use Policy:
o India will not be the first to initiate a nuclear strike, but will respond with
punitive retaliation should deterrence fail.
• Consistent with UN’s principle of right of self-defence:
o In the absence of global nuclear disarmament India's strategic interests
require effective, credible nuclear deterrence and adequate retaliatory
capability should deterrence fail. This is consistent with the UN Charter,
which sanctions the right of self-defence.
• Credible minimum nuclear deterrence.
o India shall pursue a doctrine of credible minimum nuclear deterrence. In this
policy of "retaliation only", the survivability of our arsenal is critical.
o This is a dynamic concept related to the strategic environment, technological
imperatives and the needs of national security.
o The actual size components, deployment and employment of nuclear forces
will be decided in the light of these factors.
THREE PRINCIPLES CENTRAL TO INDIA'S NUCLEAR DETERRENT
Credibility: Effectiveness: Survivability:
Any adversary The efficacy of India's nuclear forces and their
must know that India's nuclear command and control shall be
India can and deterrent be organised for very high survivability
will retaliate maximised against surprise attacks and for rapid
with sufficient through synergy punitive response.
nuclear among all They shall be designed and deployed to
weapons to elements ensure survival against a first strike and
inflict involving to endure repetitive attrition attempts
destruction and reliability, with adequate retaliatory capabilities
punishment that timeliness, for a punishing strike which would be
the aggressor accuracy and unacceptable to the aggressor.
will find weight of the Procedures for the continuity of nuclear
unacceptable if attack. command and control shall ensure a
nuclear continuing capability to effectively
weapons are employ nuclear weapons.
used against
India and its
forces.

8 | iLearn IAS
• Nuclear policy during peace time:
o India's peacetime posture aims at convincing any potential aggressor that:
▪ Any threat of use of nuclear weapons against India shall invoke
measures to counter the threat
▪ Any nuclear attack on India and its forces shall result in punitive
retaliation with nuclear weapons to inflict damage unacceptable to
the aggressor.
• No use against non-nuclear states:
o India will not resort to the use or threat of use of nuclear weapons against
States which do not possess nuclear weapons, or are not aligned with nuclear
weapon powers.
• Deterrence requires that India maintain:
o Sufficient, survivable and operationally prepared nuclear forces,
o A robust command and control system,
o Effective intelligence and early warning capabilities, and
o Comprehensive planning and training for operations in line with the strategy,
and
o The will to employ nuclear forces and weapons
• Maintaining effective conventional military capabilities:
o Highly effective conventional military capabilities shall be maintained to raise
the threshold of outbreak both of conventional military conflict as well as
that of threat or use of nuclear weapons.
• Command and Control:
o Nuclear weapons shall be tightly controlled and released for use at the
highest political level.
o The authority to release nuclear weapons for use resides in Prime Minister of
India
• Security and Safety:
o Security: Precautions shall be taken to ensure that nuclear weapons, their
manufacture, transportation and storage are fully guarded against possible
theft, loss, sabotage, damage or unauthorised access or use.
o Safety: systems shall be instituted to ensure that unauthorised or inadvertent
activation/use of nuclear weapons does not take place
o Disaster control: India shall develop an appropriate disaster control system
capable of handling the unique requirements of potential incidents involving
nuclear weapons and materials.
• Research and Development
o India should step up efforts in research and development to keep up with
technological advances in this field.

9 | iLearn IAS
o While India is committed to maintain the deployment of a deterrent which is
both minimum and credible, it will not accept any restraints on building its
R&D capability.
• Disarmament and Arms Control:
o Global, verifiable and non-discriminatory nuclear disarmament is a national
security objective. India shall continue its efforts to achieve the goal of a
nuclear weapon-free world at an early date.
o Since no-first use of nuclear weapons is India's basic commitment, every
effort shall be made to persuade other States possessing nuclear weapons
to join an international treaty banning first use.
o Having provided unqualified negative security assurances, India shall work
for internationally binding unconditional negative security assurances by
nuclear weapon states to non-nuclear weapon states.
o Nuclear arms control measures shall be sought as part of national security
policy to reduce potential threats and to protect our own capability and its
effectiveness.
o In view of the very high destructive potential of nuclear weapons,
appropriate nuclear risk reduction and confidence building measures shall be
sought, negotiated and instituted.

NEED FOR A NUCLEAR DOCTRINE:


• To bring transparency in India’s nuclear policy
• To gain trust of international community in India’s role as responsible nuclear power
• To communicate India’s nuclear retaliation capabilities to adversaries
• To re-instate our aim for a global, verifiable and non-discriminatory nuclear
disarmament
• To put in place an organized command structure for release for nuclear weapons
• To ensure strategic autonomy in decision making and protect India’s sovereign
integrity

CONCERNS:
• Increasing nuclear stock pile in the region:
o India, China expanded nuclear stockpile in last one year, according to a latest
report by Swedish think tank Stockholm International Peace Research
Institute (SIPRI).
o The report said China’s nuclear arsenal had gone up from 290 warheads in
2019 to 320 in 2020, while India’s went up from 130-140 in 2019 to 150 in
2020. Pakistan’s arsenal was estimated to be between 150-160 in 2019 and
has reached 160 in 2020.
• Sustaining threat:

10 | iLearn IAS
o Insistence of some nuclear weapons states on the legitimacy of their use
even against non-nuclear weapon countries constitutes a threat to peace,
stability and sovereignty of states.
• India’s hint of departing from No first Use (NFU) policy:
o This may risks fuelling an arms race or more unstable nuclear weapons
deployment patterns in Pakistan.
• India’s non-participation in NPT and CTBT:
o India’s stance against these international treaties creates doubts about our
nuclear disarmament goals
• Low level transparency:
o The availability of reliable information on the status of the nuclear arsenals
and capabilities of the nuclear-armed states varied considerably
o The governments of India and Pakistan make statements about some of their
missile tests but provide little information about the status or size of their
arsenals
o The U.S. had disclosed important information about its stockpile and nuclear
capabilities, but in 2019, the administration ended the practice of publicly
disclosing the size of its stockpile,
• The threat of ‘tactical’ nuclear weapons:
o In response to an alleged Indian Army
secret plan, the Cold Start Doctrine — A tactical nuclear weapon (TNW)
fast, punitive incursions into Pakistan or non-strategic nuclear weapon
territory in retaliation to terror attacks is a nuclear weapon which is
designed to be used on a
— Pakistan began to develop smaller
battlefield in military situations
‘tactical’ nuclear weapons. India does mostly with friendly forces in
not developed TNWs proximity and perhaps even on
• Dilution of bilateral nuclear arms control contested friendly territory.
agreements between Russia and the U.S: Generally smaller in explosive
o In 2019, the United States formally power, they are defined in
contrast to strategic nuclear
withdrew from the Intermediate-Range
weapons: which are designed to
Nuclear Forces (INF) Treaty be mostly targeted in the enemy
▪ INF Treaty required the United interior away from the war front
States and the Soviet Union to against military bases, cities,
eliminate and permanently towns, arms industries, and other
forswear all of their nuclear and hardened or larger-area targets
conventional ground-launched to damage the enemy's ability to
wage war.
ballistic and cruise missiles with
ranges of 500 to 5,500 km
o U.S. and Russia have reduced their nuclear arsenals under the 2010 New
Strategic Arms Reduction Treaty (New START) but it will lapse in February
2021 unless both parties agree to prolong it.

11 | iLearn IAS
▪ However, discussions to extend the New START or negotiate a new
treaty made no progress
WAY FORWARD:
• India must battle for universal elimination of all nuclear weapons to bring
sustainable global peace
• Our nuclear weapons should not be country-specific
• Our nuclear capabilities should be aimed at providing us the autonomy of exercising
strategic choices in the best interest of our country, without fear or coercion in a
nuclearized environment.
• Policy of minimum but credible deterrence should be the basic building block of
our nuclear thinking. From this, flows the decision to adopt a no-first-use posture,
and this must be maintained without change
• A cardinal principle regarding the use of nuclear weapons is that of civilian control.
Only the elected civilian leader of the country should be empowered to authorise
the use of nuclear weapons

1.5. iTips:
• For prelims, questions can be asked on Intermediate-Range Nuclear Forces (INF)
Treaty, New Strategic Arms Reduction Treaty (New START), Non-Proliferation of
Nuclear Weapons (NPT), Comprehensive Nuclear Test Ban Treaty (CTBT) etc.
• For mains, questions can be asked on Indian nuclear doctrine, India’s nuclear
weapon programme, concerns associated with it etc.

12 | iLearn IAS
2. GENE EDITING AND CRISPR Cas9
2.1 WHY IN NEWS?
Scientists Jennifer Doudna and Emmanuelle Charpentier bagged the Nobel Prize for
Chemistry for the development of one of gene technology’s sharpest tools: the CRISPR/Cas9
genetic scissors.
2.2 SYLLABUS:
GS 2: Awareness in the fields of IT, Space, Computers, robotics, nano-technology, bio-
technology
2.3 PRACTICE QUESTION:
Q. What is CRISPR-Cas9 technology. Enumerate its applications. What are the challenges
associated with it?
2.4 ANALYSIS:
WHAT IS GENE?

• A gene is a small section of DNA that contains the instructions for a specific
molecule, usually a protein.
• A chromosome contains a single, long DNA molecule, only a portion of which
corresponds to a single gene. Humans have approximately 20,000 genes arranged on
their chromosomes.
• The gene is the basic physical unit of inheritance. Genes are passed from parents to
offspring and contain the information needed to specify traits, like eye and hair
colour.
• Genes come in different forms, called alleles.
o An individual inherits two alleles for each gene, one from each parent. An
individual’s phenotype is determined by the combination of alleles they have.
o If the two alleles are the same, the individual is homozygous for that gene. If
the alleles are different, the individual is heterozygous.

13 | iLearn IAS
Phenotype refers to the observable physical properties of an organism; these
include the organism's appearance, development, and behavior. An organism's phenotype is
determined by its genotype, which is the set of genes the organism carries, as well as by
environmental influences upon these genes.
GENE, DNA AND GENOME:

GENE EDITING:
• Gene editing is a type of genetic engineering in which DNA is inserted, deleted,
modified or replaced in the genome of a living organism.
• Gene editing is performed using enzymes, particularly nucleases that have been
engineered to target a specific DNA sequence.
• These enzymes introduce cuts into the DNA strands, enabling the removal of existing
DNA and the insertion of replacement DNA.
• The most significant genome editing technologies are the Transcription Activator-
Like Effector Nucleases (TALENs), and Zinc-Finger nucleases (ZFNs) and CRISPR-Cas9.
CRISPR & Cas9:
• CRISPR technology was adapted from the natural defence mechanisms of
prokaryotic organisms like bacteria and archaea.
• Clustered Regularly Interspaced Short Palindromic A bacteriophage, also
Repeats (CRISPR) is a family of DNA sequences found in known informally as a
the genomes of the prokaryotic organisms. These pieces of phage, is a virus that
DNA are snipped off by the bacteria from viruses infects and replicates
(bacteriophages) that once attacked them. within bacteria and
• These organisms use CRISPR-derived RNA and various Cas archaea.
proteins, including Cas9, to foil attacks by viruses and
other foreign bodies.
CRISPR SCISSORS:
• In their natural form, the scissors recognise DNA from viruses, but Charpentier and
Doudna proved that they could be controlled so that they can cut any DNA
molecule at a predetermined site.

14 | iLearn IAS
• They discovered that they could “fool” the Cas9 protein by feeding it artificial RNA.
Then, the enzyme would search for anything with that same code, not just viruses,
and cut them.

SIGNIFICANCE OF CRISPR TECHNOLOGY:


• Promote gene therapeutics: The tool has enabled scientists to edit human DNA in a
lab and early-stage clinical trials are being attempted to use the tool to treat a few
diseases, including inherited disorders/diseases and some types of cancer. In the
future, it could make it easy to “rewrite the code of life”.
• Expand gene editing: The technology is not species-specific and hence can be used
on organisms previously resistant to genetic engineering. The technique is already
being explored for a wide number of applications in fields ranging from agriculture
through to human health.
• Precise editing: Unlike TALENs and Zinc-Finger Nucleases, CRISPR has a higher
degree of flexibility and accuracy in cutting and pasting DNA. Using the tool,
researchers can change the DNA of animals, plants and microorganisms with greater
precision.
• Cheaper and faster: CRISPR makes it possible to carry out genetic engineering on an
unprecedented scale at a very low cost. Also, it allows for the introduction or
removal of more than one gene at a time. This makes it possible to manipulate many
different genes very quickly, reducing the process from taking a number of years to a
matter of weeks.
SOME APPLICATIONS:
• Food preservation: The CRISPR/Cas 9 system was first exploited by a food processing
company to improve the immunity of bacterial cultures against viruses. Today, many
food manufacturers now use the technology to produce cheese and yoghurt.

15 | iLearn IAS
• Treating genetic disorders: Scientists have reported the successful use of CRISPR/Cas
9 in laboratories in treating muscular dystrophy, thalassemia, sickle cell disease and
in making human cells immune to HIV.
• Organ cultivation: CRISPR is being investigated in conjunction with pluripotent stem
cells for the creation of transgenic animals to produce organs for transplants into
human patients.
• Vector control: The technology is also being investigated as a means to genetically
engineer insects so as to wipe out insect-borne diseases such as malaria, transmitted
by mosquitoes, and lyme disease, transmitted by ticks.
• COVID-19 control: The CSIR-Institute of Genomics and Integrative Biology (CSIR-IGIB)
in Delhi developed a COVID-19 paper strip testing kit, nicknamed ‘Feluda’, based on
the CRISPR/Cas9 system.
CONCERNS:
• Grey areas: The technology is still in its infancy and knowledge about the genome
remains very limited. Many scientists caution that the technology still needs a lot of
work to increase its accuracy and prevent side effects.
• Unforeseen outcomes: Gene editing can inadvertently wipe out and rearrange large
swaths of DNA in ways that may imperil human health. This was supported by recent
studies showing that CRISPR-edited cells have inadvertently triggered cancer in some
cases.
• Ethical concerns: Its questionable use has been a matter of controversy. The most
controversial application was in 2018, when a Chinese researcher used it to create
‘gene-edited twins’ via in-vitro fertilisation. The desired mutations were not
achieved, and there were a host of other unintended mutations too.
• Philosophical dilemma: Several religious philosophies and beliefs are against
researches in germline manipulations. Also, while it will take many more years
before the technology will be viable to use to create ‘designer babies’, a public
debate has already begun on this issue.
• Unclear regulatory regime: Policy-makers are still debating about what limitations to
put on the technology. The US National Institutes of Health has stated that it will not
fund any research that uses genome editing tools such as CRISPR in human embryos.
Meanwhile, the UK's Authority has allowed its use on human-animal hybrid embryos
under 14 days old.
• Threat to ecology: Once an organism, such as a plant or insect, is modified, they are
difficult to distinguish from the wild-type and once released into the environment
could endanger biodiversity.
CONCLUSION:
The system purportedly holds promise for treating more complex diseases, such as
cancer, heart diseases, mental illnesses, and HIV infection and researches are already
underway in this direction. However, it is yet to reach the level of precision required to be

16 | iLearn IAS
applicable in a massive scale. Hence, the technology needs a dynamic and effective
regulatory regime that encourages research and investment, but at the same time maintains
ethical and legal boundaries.
2.5. iTips:
• For Prelims, questions on fundamental principles of DNA, genome, gene editing,
CRISPR etc. can be asked.
• For mains, questions on the basics of gene editing and CRISPR technologies, thier
applications and concerns can be asked.

17 | iLearn IAS
3. LANDMARK JUDGMENTS PASSED BY THE SUPREME
COURT IN 2019
3.1 WHY IN NEWS?
The Supreme Court delivers some crucial judgments in 2019, which shape the legal system
of India.
3.2 SYLLABUS:
GS 2: Indian Constitution—Historical Underpinnings, Evolution, Features, Amendments,
Significant Provisions and Basic Structure.
GS 2: Structure, Organization and Functioning of the Executive and the Judiciary
3.3 PRACTICE QUESTION:
Q. Judiciary has played an important role in expanding the scope of Right to Information Act
(RTI).Elucidate?
3.4 ANALYSIS:
DIVESTMENT OF CBI’S DIRECTOR
• What was the case?
o The Central Vigilance Commission (CVC), in October 2018, took away the
powers and functions of Mr. Alok Kumar Verma, Director of Central Bureau of
Investigation under the Prevention of Corruption Act, 1988.
o Thereafter a writ petition was filed challenging the validity of the said order
and the petitioners argued that the order passed by the Central Government
was violative of Section 4 of Delhi Special Police Establishment (DSPE) Act
and the Supreme Court guidelines issued in Vineet Narain case.
o Under Section 4A of the DSPE Act, the approval of the Selection Committee
is necessary to divest the powers of the director of CBI.
• Court ruling:
o In January 2019, the Supreme Court held that the orders issued by the
Central Government were not valid and thus quashed them.
o Alok Verma was reinstated with his powers and duties.

CONSTITUTIONAL VALIDITY OF INSOLVENCY AND BANKRUPTCY CODE UPHELD


• What was the case?
o The constitutional validity of the IBC and National Company Law Tribunal has
been questioned
o Petitioners argued that difference between operational creditors and
financial creditors were violative of Article 14 of the Constitution
• Court ruling:
o Supreme Court, in Swiss Ribbons Pvt Ltd. v Union of India, held that the
difference between operational creditors and financial creditors were based

18 | iLearn IAS
on intelligible differentia and thus not violative of Article 14 of the
Constitution.
o Supreme Court upheld the constitutional validity of Insolvency and
Bankruptcy Code, 2016 and NCLT.

CINEMA BAN IN WEST BENGAL OVERTURNED:


• What was the case?
o Bengali film “Bhobishyoter Bhoot” was stopped from screening by the Bengal
Government in most of the cinemas.
o The Government contended that the movie was politically sensitive and it
may hurt some sentiments and cause disorder in the State
o A plea was filed by the producers of the film claiming their fundamental right
to speech and expression has been violated
• Court ruling:
o Supreme Court held that Public officials and the State Government are
subject to the rule of law and cannot gag free speech due to fear of violence.
o The ban imposed on the Bengali film was overturned and compensation was
provided to the producers.
Nagraj case,2006
In this case, the Supreme Court had
RESERVATION IN PROMOTIONS UNDER
held that it is not mandatory for the
KARNATAKA ACT UPHELD State to make reservations for
• What was the case? scheduled castes or scheduled tribes
o Karnataka in 2002, enacted a law (SC/STs) in matter of promotions.
that stated consequential However, if the State wishes to
seniority would be applicable exercise its discretion, it has to gather
while dealing with promotions of quantifiable data showing
backwardness of the class and
the SC/ST employees in
inadequacy of representation of that
government offices. class in public employment.
o This implied that a reserved For making reservations in promotion,
category employee could be states also need to comply with the
promoted before a senior requirement of maintaining
employee belonging to the administrative efficiency as per Article
335.
general category.
It was further held that the State is
o In 2007 the Supreme Court held required to adhere to the ceiling-limit
that the law passed by the of 50 percent and abstain from
Karnataka Government did not obliterating the creamy layer or
comply with the guidelines extending the reservation indefinitely.
established under the Nagraj case and was thus unconstitutional.
o Subsequently, the Karnataka Government set up a Committee to
demonstrate the criteria laid down in the Nagraj case has been fulfilled, i.e.
▪ (1) current backwardness of SC/ST

19 | iLearn IAS
▪ (2) inadequate representation and
▪ (3) and impact on administrative efficiency
o And thereafter re-enacted the earlier law.
o The constitutional validity of the re-enacted law was questioned.
o The Supreme Court held that the deficiency that was noted in the 2002 Act
have been fulfilled and the Reservation Act 2018 is thereby valid under
Article 16(4A) of the Constitution.
• Court ruling:
o The constitutional validity of the Karnataka Act approving consequential
seniority in promotions for the reserved category was upheld.

GOVERNMENT-FUNDED NGOS COME UNDER RTI:


• What was the case?
o While deciding an appeal filed by the DAV management wherein they
contended that they are not public authorities and thus do not come under
Right to Information Act, the Court held that public authorities defined
under Section 2(h) of the Right to Information Act, 2005 would include all
such institutions which are ‘substantially’ financed by the Government.
o Substantially financing does not mean that the government should finance
50% or more, but it means that a large portion is financed by the
government and the finance can be either direct or indirect.
o This means that authorities that are financed by the government including
the NGOs will have to maintain records as prescribed in the Act and vital
information has to be presented to every citizen who asks for the same.
o The Supreme Court added that every citizen has a right to know where the
money is being spent by their government, thus NGOs financed by the
government should be transparent in its proceedings and records.
• Court ruling:
o NGOs which are directly or indirectly, substantially financed by the
government come under the RTI Act and thus every citizen has the right to
ask for information from them.

VERDICT ON AYODHYA- BABRI MASJID CASE:


• What was the case?
o The Ayodhya- Babri Masjid dispute is the longest property dispute in the
history of India.
o The case was finally concluded by the Supreme Court after 134 years from
the first case filed on this matter.
o The Allahabad High Court delivered a judgment in 2010 wherein the land in
dispute was divided into three equal parts, the judgment did not satisfy any
of the parties involved and thus an appeal was filed in Supreme Court.

20 | iLearn IAS
o After 9 years and 40 days of continuous hearing, the Court gave one of the
most crucial judgments.
o The Court after observing the developments of the case scrapped the High
Court verdict and held that the land in dispute is to be awarded to the Hindu
Deity ‘Ram’ for the construction of the temple.
o It was observed that the land was not of Islamic origin and the Masjid was
not built on vacant land.
o The Court also ordered that a suitable alternative land of 5 acres is to be
allotted to the Sunni Waqf Board for construction of the mosque in Ayodhya
itself.
• Court ruling:
o The disputed land was given to Ram Lalla and the Central Government has
been ordered to formulate a scheme and set up a trust within 3 months for
the construction of the temple.

CHIEF JUSTICE OF INDIA COMES UNDER RTI:


• What was the case?
o In November 2019, the Supreme Court in its historic judgment held that the
CJI comes under the Right to Information Act and is a public authority under
Section 2(h) of the Act.
o This implies that the CJI is to be transparent and is answerable to all
questions raised by the citizens of the County.
o However, the court also emphasized the importance of maintaining
confidentiality under certain aspects of the judiciary’s working.
o The RTI will apply to CJI only when it is in the interest of the public and does
not hamper the proceedings of the judiciary in any manner.
• Court ruling:
o CJI is a public authority under the RTI Act.

3.5. iTips:
• For prelims, questions can be asked on Article 16 of the Constitution, Salient
provisions of RTI, IBC etc.
• For mains, this input can be utilized while answering questions related to structure,
organization and functioning of the executive and the judiciary, evolution of Indian
Constitution etc.

21 | iLearn IAS
4.NATIONAL COMMISSION FOR PROTECTION OF CHILD RIGHTS
4.1 WHY IN NEWS?
The Supreme Court sought a response from the National Commission for Protection
of Child Rights (NCPCR), on the commission’s request to eight States to repatriate children
living in care homes with their families.
4.2 SYLLABUS:
GS 2: Statutory, regulatory and various quasi-judicial bodies.
4.3 PRACTICE QUESTION:
Q. Elaborate the mandate and functions of the National Commission for Protection of Child
Rights (NCPCR). Is the Commission able to strategize and tackle the problems that children
face in cyber space. Comment?
4.4 ANALYSIS:
NATIONAL COMMISSION FOR PROTECTION OF CHILD RIGHTS:
• The National Commission for Protection of Child Rights (NCPCR) is the apex body for
upholding, monitoring and facilitating child rights in the country.
• It is a statutory body, set up in 2007 under the Commissions for Protection of Child
Rights (CPCR) Act, 2005.
• It functions under the administrative control of the Ministry of Women & Child
Development, Government of India.
STRUCTURE:
The commission consist of:
• A chairperson who is a person of eminence and has done outstanding work for
promoting the welfare of children; and
• Six members, out of which at least two are woman, from the following fields,
is appointed by the Central Government from amongst person of eminence, ability,
integrity, standing and experience in:
1. Education
2. Child health, care, welfare or child development
3. Juvenile justice or care of neglected, marginalized or children with disabilities
4. Elimination of child labour or children in distress
5. Child psychology or sociology; and
6. Laws relating to children
MANDATE:
• To ensure that all Laws, Policies, Programmes, and Administrative Mechanisms are in
consonance with the Child Rights perspective as enshrined in the Constitution of
India and also the UN Convention on the Rights of the Child.
• The Child is defined as a person in the 0 to 18 years age group.

22 | iLearn IAS
MAJOR FUNCTIONS AND POWERS:
1. Examine and review the safeguards for the protection of child rights and
recommend measures for their effective implementation.
2. Inquire into violation of child rights and recommend initiation of proceedings in
such cases. The body has quasi-judicial powers whereby they can investigate,
summon and recommend cases to the courts, but cannot pass judgments and hand
out punishments.
3. Study treaties and other international instruments and undertake periodical
review of existing policies, programmes and other activities on child rights and make
recommendations for their effective implementation.
4. Undertake and promote research in the field of child rights and compile and analyse
data on children.
5. Spread child rights literacy among various section of society through publications,
the media, seminar and other available means and promote the incorporation of
child rights into the school curriculum, training of teachers or personnel dealing with
children.
6. Inspect any juveniles custodial home, or places where children are detained or
lodged for the purpose of treatment, reformation or protection, under the control of
governments or other authorities and take up with these authorities for remedial
action, if found necessary.
7. Examine all factors that inhibit the enjoyment of rights of children affected by
terrorism, communal violence, riots, natural disaster, domestic violence, HIV/AIDS,
trafficking, maltreatment, torture and exploitation, pornography and prostitution
and recommend appropriate remedial measures.
8. Look into the matters relating to the children in need of special care and protection
including children in distress, marginalized and disadvantaged children, children in
conflict with law, juvenile children without family and children of prisoners and
recommend appropriate remedial measures.
9. Inquire into complaints and take Suo motu notice of matter relating to violation of
child rights, non-implementation of laws, non-compliance of policy decisions etc.
10. Undertake formal investigation where concern has been expressed either by
children themselves or by concerned person on their behalf.
11. Present reports upon working of safeguards to the central government, annually and
at such other intervals as the Commission may deem fit.
12. Such other functions as it may consider necessary for the promotion of Child Rights.
RESPONSIBILITIES UNDER OTHER ACTS:
National and state commission for Protection of Child Rights (NCPCR) have been
created to oversee the implementation of the Juvenile Justice Act in the country. Besides
this, the commission also has responsibilities under two other acts:

23 | iLearn IAS
1. To monitor the implementation of Protection of Children from Sexual Offences
(POCSO) Act, 2012.
2. Examine and review the safeguards provided under Right to Information Act, 2005
and inquire into complaints relating to child's right to free and compulsory
education.
MAJOR INITIATIVES BY THE COMMISSION:
• POCSO e-box: It is an online online complaint management system enables easy
reporting and timely action against the offenders under the POCSO Act, 2012. E-Box
is very simple to operate and will help to maintain the confidentiality of the
complaint.
• Parkisha Parv 2.0: the second edition of a month-long campaign for stress-free
examinations. It provided a platform to students to talk about their concerns and
interact with renowned counsellors and psychologists.
• 24x7 helpline 1098, for children in distress in operation through Childline India
Foundation.
• Quick response cell for rescue of trafficked children or sexual abuse cases or stop
instances of child marriage.
• Guidelines for Eliminating Corporal Punishment in Schools. It calls for schools to
constitute special monitoring cells to take prompt action in cases of physical
punishment or harassment of children.
• Standard Operating Procedures for the Care, Protection and Rehabilitation of
Children in various conditions, such as children in conflict with law, sexually abused,
Runaway etc.
• Handbook for Media Professionals and guidelines for certification bureaus to avoid
portrayal of violation of child rights.
ISSUES:
• Weak inspections: The safety and security of child care homes have been a matter of
concern ever since sexual assaults were reported in child care institutions in Deoria
in Uttar Pradesh and Muzaffarpur in Bihar in 2018. This arise due to the poor
regulation of such institutions by the NCPCR.
• Political interference: It often seems to be deriving its priorities from the political
agenda of the day and being a tool for the witch-hunt of non-governmental agencies
critical of the government. Eg: In the Shaheen Bagh protests, the commission
threatened action against the children of the women protesting there based on
anonymous complaint.
• Concerns over recent actions: Most of the children are in CCIs due to abusive
conditions in the family, and a mandated repatriation within a short period of time
would likely place the children again at grave risk of abuse, exploitation and neglect.
• Deficit infrastructure: There is a severe lack of child care institutions (CCI) in India,
particularly in the northeastern states. As a result, the number of children residing in

24 | iLearn IAS
these CCIs are in excess, which pose a potential risk to the rights and protection of
these children.
• Inadequate funding: The commission is funded by grants from both central and
state governments. However, this financial assistance provided to the Commission is
very less to cater to its needs.
• No enforcement power: The body is only recommendatory and has no power to
enforce its decisions. The body has often been reactive and arbitrarily taken up
select issues and responded to them.
CONCLUSION:
The pandemic has exacerbated existing issues of child malnutrition, child labour, child
abuse, child marriage and mental illness. This situation reiterates the need for strengthening
all child-related institutions in the country. To cater to this need and to uphold its steadfast
and fair commitment to the welfare of children, the CCIs needs monitoring and reforms.
4.5. iTips:
• For Prelims, questions on the structure, functions and initiatives of the commission
can be expected.
• For mains, the content can also be used in questions related to vulnerability faced by
children and child rights issues.

25 | iLearn IAS
5. GORKHALAND ISSUE
5.1 WHY IN NEWS?
The Home Ministry had convened a meeting with the Gurkha Janmukti Morcha
(GJM) on 8th Oct 2020. The group demanded the creation of a State of Gorkhaland
comprising Darjeeling and the adjoining hill districts in West Bengal.
5.2 SYLLABUS:
GS 1: Regionalism
GS 3: Linkages between Development and Spread of Extremism. Role of External State and
Non-state Actors in creating challenges to Internal Security.
5.3 PRACTICE QUESTION:
Q.“The demand for a separate state of Gorkhaland has often been weighed by reason and
also by emotion”. In the light of this statement analyse the role of cultural distinctiveness in
various separatists’ movements across India?
5.4 ANALYSIS:
WHAT IS GORKHALAND ISSUE?
• Gorkhaland consists of Nepali-speaking people of Darjeeling, Kalimpong, Kurseong
and other hilly districts of West-Bengal.
• The people belonging to these areas have ethical, cultural and language differences
with the Bengali community of West-Bengal.
• The demand of Darjeeling as a separate administrative region dates back to 1907.
• But, the term “Gorkhaland” was coined recently, in the 1980s, by Subhash Ghising,
the founder of Gorkha National Liberation Front (GNLF).
• The Gorkhaland Movement is a movement mainly focused in the Darjeeling Hills of
West Bengal, which demands the creation of a separate state of Gorkhaland.
• The area covers Duars and Terai region of West Bengal. And is famous for its tea and
beauty, which are the main sources of its income.

REASONS FOR THE SEPARATIST DEMAND:


• Cultural differences:
o People of Nepali-Indian Gorkha ethnic origin on the Northern part of West
Bengal demands a state on basis of their cultural identity, which is very
different from Bengali culture.
• Backwardness:
o Improvements in basic amenities like education, health, water supply,
sanitation and employment is stagnated in the hill regions
• Cultural imposition:
o In 2017, it was West Bengal government’s decision to impose Bengali
language in all the schools from Class I to IX, attracts violent agitation in the
region

26 | iLearn IAS
• Political aspirations:
o State government’s failure to conduct the Darjeeling Gorkha Hill Council
(DGHC) elections created mistrust among the public and rose their aspiration
of self-governance

HISTORY OF THE MOVEMENT:


• Prior to 1780s the Darjeeling area was ruled by Chogyal of Sikkim
• Around 1780 the Gorkhas had captured Sikkim and most parts of the North East
including Darjeeling.
• In 1817, through the Treaty of Titalia, the British reinstated Chogyal of Sikkim and
gave back all the territory annexed by Gorkhas back to Chogyals.
• Though in 1835, British took possession of the hills of Darjeeling from Sikkim.
• The present district of Darjeeling came into existence in 1866.
• In 1907, The demand for a separate administrative unit in Darjeeling was raised for
the first time by the Hillmen’s Association of Darjeeling. It submitted a
memorandum to Minto-Morley Reforms demanding a separate administrative
setup
• In 1781, The West Bengal government passes a resolution supporting the creation of
an autonomous district council consisting Darjeeling and related areas.
• Gorkha National Liberation Front (GNLF) was formed in 1980, it launches a most
violent agitation in Gorkhaland movement history in 1986.
• In 1988, Darjeeling Gorkha Hill Council accord is signed by GNLF, the state of Bengal
and the Centre. Darjeeling Gorkha Hill Council came into action. GNLF drops the
demand for the separate state.
• In 2005, the same parties signed an in–principle memorandum of settlement to
include Darjeeling in the Sixth Schedule of the Indian Constitution, which addresses
the administration of tribal areas.
• Calling ‘the Sixth Schedule solution’ a betrayal to Gorkhaland, Bimal Gurung
launched Gorkha Janmukti Morcha (GJM) in 2007.
• The same year witnessed a rise in agitations for the separate Gorkhaland demand.
• In 2011 the memorandum of agreement for the formation of a Gorkhaland
Territorial Administration (GTA), a semi-autonomous administrative body for the
Darjeeling, passed by West Bengal legislative assembly to calm the GJM.
• In 2013, the demand for separation rose high again due to the division of Andhra
Pradesh
• In 2017, West Bengal government’s decision to impose Bengali language in all the
schools from Class I to IX, had triggered new set of violence in the region
SIGNIFICANCE OF THE MOVEMENT:
• Affects India-Nepal relation:
o Gorkhaland will affect India-Nepal relations too.

27 | iLearn IAS
o How India treats the problems of the Nepali people of hill origin in Darjeeling
will affect how Nepal deals with the people of Indian origin in Nepal, i.e.
Madhesi in Terai.
• Strategic location:
o Gorkhaland has a strategic location, it’s vicinity to the chicken neck that
connects rest of India with North East. Its stability is must for India’s strategic
and economic interests of the nation.
• Ensuring stability in the region:
o Darjeeling is a tea and tourist hot-spot with a high level of poverty. It needs
and has potential to become the economic engine of the East with a
sustainable economic model. But such things will be possible only if there is
stability in the region.
• Influences other separatist movements across the country
o The way government dealing with Gorkhaland issue will affect the other
similar demands such as Saurashtra, Bodoland etc
• A Gorkhaland State Would Open a Pandora's Box:
o Any consideration of the demand would open up a pandora’s box with many
ethnic and indigenous groups raising their demands for statehood in the
region.

WAY FORWARD:
• Solution through political consensus:
o Creation of separate state may legitimise violence as a way to meet
demands.
o The rise of agitation with the rise of new outfits shows that politics plays the
vital role.
o The problem can be resolved by consensus among various stakeholders
including state government, separatists and Central government.
• More powers to Gorkhaland Territorial Administration
o Giving more powers to Gorkhaland Territorial Administration, creating a
transparent and open election process. It must be realised that GJM rose due
to the failure of the government to conduct DGHC elections.
• Economic development in the region:
o Improving infrastructure for tourism and tea sector can bring employment
opportunities and tackle poverty in the region. This may help to pacify people
of the region
• Cultural assimilation:
o Respecting the distinct cultural and linguist aspirations of the Gorkhaland
people is required for assimilating them with main land Bengal
o Regressive moves such as imposing Bengali language as done in 2017, may
prove disastrous

28 | iLearn IAS
• Tribal status:
o Granting tribal status to the 11 communities of the Darjeeling hills, as
demanded by Gorkha Janmukti Morcha (GJM)

5.5. iTips:
• For prelims, questions can be asked on the Gorkhaland issue, factors driving demand
for separate state, similar movements across India, linkages between development
and spread of extremism etc.
• For mains, map based questions are expected on the location of Darjeeling and
adjacent areas, provisions of Schedule 6 of the Constitution etc.

29 | iLearn IAS
WEEKLY PRELIMS COMPILATION

30 | iLearn IAS
Nitrous Oxide
Why in news?
• Human emissions of nitrous oxide (N2O) increased by 30 per cent between
1980 and 2016, according to a research paper published in Nature
About Nitrous Oxide:
• N2O is a greenhouse gas 300 times more potent than carbon dioxide (CO2).
• It has the third-highest concentration — after CO2 and methane — in our
atmosphere among greenhouse gases responsible for global warming.
• N2O can live in the atmosphere for up to 125 years.
• Its global concentration levels increased from 270 parts per billion (ppb) in
1750 to 331 ppb in 2018 — a jump of 20 per cent. The growth has been the
quickest in the past five decades because of human emissions.
• It is estimated that 30% of the N2O in the atmosphere is the result of
human activity, chiefly agriculture and industry.
• Nitrous oxide occurs in small amounts in the atmosphere, but has been
found to be a major destroyer of stratospheric ozone, with an impact
comparable to that of CFCs.
• The research was conducted through an international collaboration
between the International Nitrogen Initiative (INI) and the Global Carbon
Project of Future Earth, a partner of the World Climate Research
Programme.
PRELIMS QUESTION:
1.Consider the following statements regarding Nitrous Oxide:
1. It is both an ozone depleting substance and a greenhouse gas.
2. Nitrous oxide does not occur naturally and emissions are completely
anthropogenic.
Which of the statements given above is/are correct?
(a) 1 only
(b) 2 only
(c) Both 1 and 2
(d) Neither 1 nor 2
Answer: A

Red Sea
Why in news?
• The Saudi-Led coalition fighting in Yemen foiled an “imminent terrorist
attack” by Iran-aligned Houthis in the south of the Red Sea, as per reports.

31 | iLearn IAS
About Red Sea:
• Red Sea is a seawater inlet of the Indian Ocean, lying between Africa and
Asia.
• The connection to the ocean is in the south through the Bab el Mandeb
strait and the Gulf of Aden.
• The sea separates the coasts of Egypt, Sudan, and Eritrea to the west from
those of Saudi Arabia and Yemen to the east.
• To the north lie the Sinai Peninsula, the Gulf of Aqaba and the Gulf of Suez
(leading to the Suez Canal).
• Red sea is connected to Mediterranean Sea via Suez Canal.
• Normally, the Red Sea is an intense blue-green; occasionally, however, it is
populated by extensive blooms of the algae Trichodesmium erythraeum,
which, upon dying off, turn the sea a reddish-brown colour.

PRELIMS QUESTION:
2.Which of the following countries have a coast on the red sea:
1. Saudi Arabia
2. Egypt
3. Jordan
4. Kuwait
Select the correct answer using the code given below:
(a) 1,2 and 3 only
(b) 1 and 2 only
(c) 1,2 and 4 only
(d) 1,2,3 and 4
Answer: A

32 | iLearn IAS
GST compensation
Why in news?
• The GST Council meeting held Monday to discuss the compensation of
states for the shortfall in GST collections again ended without reaching any
consensus.
• The panel, which is the highest decision-making body on indirect taxes,
failed to arrive at an agreement on the Centre’s proposal of states
borrowing against future GST collections to make up for the shortfall.
Why is the Centre required to compensate states for GST?
• With GST implementation in 2017, the principle of indirect taxation for
many goods and services changed from origin-based to destination-based.
• This means that the ability to tax goods and services and raise revenue
shifted from origin states (where the good or service is produced) to
destination states (where it is consumed).
• This change posed a risk of revenue uncertainty for some states and their
concern was addressed through constitutional amendments, requiring
Parliament to make a law to provide for compensation to states for five
years to avoid any revenue loss due to GST.
About the compensation:
• For this purpose, the GST (Compensation to States) Act was enacted in
2017 on the recommendation of the GST Council.
• The Act guarantees all states an annual growth rate of 14 percent in their
GST revenue during the period July 2017-June 2022.
• If a state’s GST revenue grows slower than 14 percent, such ‘loss of
revenue’ will be taken care of by the Centre by providing GST
compensation grants to the state.
• To provide these grants, the Centre levies a GST compensation cess on
certain luxury and sin goods such as cigarettes and tobacco products, pan
masala, caffeinated beverages, coal, and certain passenger vehicles.
• The Act requires the Centre to credit this cess revenue into a separate
Compensation Fund and all compensation grants to states are required to
be paid out of the money available in this Fund.
How much compensation is provided to states?
• For 2018-19, Centre gave Rs 81,141 crore to states as GST compensation.
• However, for the year 2019-20, the compensation requirement of states
nearly doubled to Rs 1.65 lakh crore.
• This can be attributed to the economic slowdown which resulted in a lower
GDP growth and thereby lower growth in tax collections which grew by
just 4 percent.
33 | iLearn IAS
• Subsequently states had to be compensated by the centre. But large parts
of the compensation were delayed, almost by an year.
What led to a delay in payment of compensation to states?
• In 2019-20, the delay in payment was observed due to insufficient funds
with the Centre for providing compensation to states.
• The sale of goods taxed for the compensation did not provide necessary
funds as sales of some of them were affected by the economic slowdown.
• The shortfall in collections for 2019-20 was met through:
o Surplus cess collections from previous years
o Partial cess collections of 2020-21
o A transfer of Rs 33,412 crore of unsettled GST funds from the Centre
to the Compensation Fund. These unsettled funds are GST
collections, generated in 2017-18 from interstate and foreign trade,
that have not yet been settled between centre and states.
Scenario in 2020-21
• In the 2020-21 budget, the Centre has estimated a 10 percent growth in
nominal GDP. However, due to the impact of COVID-19 and the lockdown,
the actual growth in 2020-21 is likely to be much lower.
• In such a scenario, states’ GST revenue would also be much lower than
expected, thus leading to a higher compensation requirement.
• However, the ability of the Centre to pay compensation depends on the
cess collections, which are also getting impacted this year.
• Under the GST (Compensation to States) Act, 2017, Centre can provide
compensation to states only through the money available in the
Compensation Fund.
• Despite a shortfall of money in the Compensation Fund, the Centre is
constitutionally obligated to meet states’ compensation requirement for a
period of five years.
• Six states Delhi, Gujarat, Karnataka, Maharashtra, Punjab, and Tamil Nadu
accounted for 52 percent of the total requirement of compensation for
2019-20.
• The Centre had in August given two options to the states
o To borrow either Rs 97,000 crore from a special window facilitated by
the RBI or Rs 2.35 lakh crore from the market.
o It had also proposed extending the compensation cess levied on
luxury, demerit and sin goods beyond 2022 to repay the borrowing.
• But most non-BJP ruled states have rejected the demand and asked the
centre to borrow the amount instead of the states.
PRELIMS QUESTION:
3.Consider the following statements regarding GST compensation fund:
34 | iLearn IAS
1. Centre is obligated to pay GST compensation to states for ten years in the
event of a short fall in GST collections.
2. GST (Compensation to States) Act 2017 guarantees all states an annual
growth rate of 14 percent in their GST revenue, failing which they will be
compensated.
Which of the statements given above is/are correct?
(a) 1 only
(b) 2 only
(c) Both 1 and 2
(d) Neither 1 nor 2
Answer: B

Rashtriya Kamdhenu Aayog


Why in news?
• The Rashtriya Kamdhenu Aayog unveiled a “chip” which has been made of
cow dung which they claim would significantly reduce the radiation
generated from mobile handsets.
About Rashtriya Kamdhenu Aayog:
• Rashtriya Kamdhenu Aayog is an apex advisory body to address the issues
related to conservation, protection and development of cows and their
progeny.
• It was created by a government resolution in February 2019 and will
function as an integral part of Rashtriya Gokul Mission.
Objectives
• To organize animal husbandry on modern and scientific lines
• To address the issues related to and to take steps for preserving and
improving breeds.
• To provide policy and direction to the cattle conservation and development
programmes in the country.
• To ensure proper implementation of laws with respect to prohibition of
slaughter and / or cruelty to cows.
• It will particularly focus on development of appropriate programmes for
conservation, sustainable development and genetic upgradation of
Indigenous breeds of cows.
• It also aims at transmission and application of improved technology and
management practices at the farmers’ doorstep through coordination with
dairy cooperatives, farmer producer companies and dairy industry as well
as research institutions.

35 | iLearn IAS
Cattle in India:
• As per 2012 livestock census, India has 300 million bovine population; out
of this 191 million are cattle (cows) and 108.7 million buffaloes.
• The cattle and buffalo genetic resource is constituted by 43 breeds of cattle
and 16 breeds of buffaloes.
• India is the global leader in milk production with peak milk production of
176.35 MMT during 2017-18.
PRELIMS QUESTION:
4.Consider the following statements regarding Rashtriya Kamdhenu Aayog:
1. It is a statutory Body.
2. One of its objectives is genetic upgradation of Indigenous breeds of cows.
3. It was created in 2014 by government of India.
Which of the statements given above is/are correct?
(a) 1 and 2 only
(b) 2 only
(c) 2 and 3 only
(d) 1,2 and 3
Answer: B

The Human Cost of Disasters report


Why in News
• “The Human Cost of Disasters 2000-2019” report has been published by
UNDRR.
About the report:
• The report is published by the United Nations Office for Disaster Risk
Reduction (UNDRR).
• It said that 7,348 major disaster events had occurred between 2000 and
2019, far more than the 4,212 major natural disasters recorded between
1980 and 1999.
• The sharp increase was largely attributable to a rise in climate-related
disasters, including extreme weather events like floods, drought and
storms
• The report did not touch on biological hazards and disease-related
disasters like the coronavirus pandemic, which has killed over one million
people and infected over 37 million in the past nine months.
• The data showed that Asia has suffered the highest number of disasters in
the past 20 years with 3,068 such events, followed by the Americas with
1,756 and Africa with 1,192.

36 | iLearn IAS
• The deadliest single disaster in the past 20 years was the 2004 Indian
Ocean tsunami, with 2,26,400 deaths, followed by the Haiti earthquake in
2010, which claimed some 2,22,000 lives.
• In terms of affected countries, China topped the list with 577 events
followed by the United States with 467 while India had 321.
United Nations Office for Disaster Risk Reduction
• The United Nations Office for Disaster Risk Reduction (UNDRR) was created
in December 1999 to ensure the implementation of the International
Strategy for Disaster Reduction.
• UNDRR (formerly UNISDR) is part of the United Nations Secretariat and it
supports the implementation & review of the Sendai Framework for
Disaster Risk Reduction adopted by the Third UN World Conference on
Disaster Risk Reduction.
• The Sendai Framework is a 15-year voluntary people-centred approach to
disaster risk reduction, succeeding the 2005-2015 framework.
• UNDRR is headquartered in Geneva, Switzerland.
PRELIMS QUESTION:
5.The Human Cost of Disasters report is published by
(a) UNESCO
(b) United Nations Office for Disaster Risk Reduction
(c) World Economic Forum
(d) UN HABITAT
Answer: B

Mega Food Park Scheme


Why in News?
• First Mega food park in Kerala was recently inaugurated.by union minister
for food processing.
Objective:
• The primary objective of the Scheme is to provide modern infrastructure
facilities for the food processing along the value chain from the farm to the
market with a cluster-based approach based on a hub and spokes model.
Features:
• It is part of the Central Sector Scheme – Pradhan Mantri Kisan SAMPADA
Yojana (Scheme for Agro-Marine Processing and Development of Agro-
Processing Clusters).
• Ministry of Food Processing Industries implements the scheme.
• The scheme aims at creation of infrastructure for primary processing and
storage near the farm in the form of Primary Processing Centres (PPCs) and
37 | iLearn IAS
Collection Centres (CCs)and common facilities and enabling infrastructure
like roads, electricity, water, ETP facilities etc. at Central Processing Centre
(CPC).
• The financial assistance under the scheme is provided in the form of grant-
in-aid at 50%of eligible project cost in general areas and at 75% of eligible
project cost in NE Region and difficult areas.
• This is implemented via a Special Purpose Vehicle where the food
processing company will have a minimum equity of 26%.

PRELIMS QUESTION:
6.Consider the following statements regarding mega food parks scheme:
1. It is under the ministry of agriculture.
2. It is part of the umbrella scheme Krishonnati Yojana.
Which of the statements given above is/are correct?
(a) 1 only
(b) 2 only
(c) Both 1 and 2
(d) Neither 1 nor 2
Answer: D

Tiger
Why in news?
• A picture of a tigress hugging a tree has won the top prize at the prestigious
Wildlife Photographer of the Year 2020 awards
About Tigers:
• The tiger (Panthera tigris) is the largest extant cat species and a member of
the genus Panthera.
38 | iLearn IAS
• Around 9 subspecies are reported most important of which are Sumatran
Tiger, Amur (or Siberian) tiger, Indian (or Bengal) tiger etc.
• Some sub species like Bali tiger are believed to be extinct.
• India is home to the world's largest population of wild tigers with estimated
2967 tigers in 2019, followed by Russia.
• The tiger is the national animal of India, Bangladesh, Malaysia and South
Korea.
Habitat and character:
• It is an apex predator who is territorial and generally a solitary but social
predator.
• Tigers require large contiguous areas of habitat, which support its
requirements for prey and rearing of its offspring.
• There is a notable sexual dimorphism between male and female tigers, with
the females being consistently smaller.
• When not subject to human disturbance, the tiger is mainly diurnal ie
active in the day. But nocturnal hunting is also observed regularly.
• Tiger is a strong swimmer and often bathes in ponds, lakes and rivers, thus
keeping cool in the heat of the day.
PRELIMS QUESTION:
7.Consider the following statements regarding Tigers:
1. They are the largest extent cat species.
2. India has the largest population of wild tigers in the world.
Which of the statements given above is/are correct?
(a) 1 only
(b) 2 only
(c) Both 1 and 2
(d) Neither 1 nor 2
Answer: C

Anti-defection law
Why in news?
• Bypolls to a legislative assembly seat in Bengaluru will be held soon.
• The seat has been vacant since the sitting MLA, was disqualified under the
anti-defection law last year after crossing from the Congress to the BJP.
What is Anti-defection law?
• The Act provided for the disqualification of the members of Parliament
and the state legislatures on the ground of defection from one political
party to another.

39 | iLearn IAS
Introduced in?
• In 1985, through the 52nd Amendment Act, the government had included
the Tenth Schedule or the Anti-Defection Act in the Constitution.
Provisions
• Disqualification of Members of Political Parties: A member of a House
belonging to any political party becomes disqualified for being a member of
the House
• if he voluntarily gives up his membership of such political party; or
• if he votes or abstains from voting in such House contrary to any direction
issued by his political party without obtaining prior permission of such party
and such act has not been condoned by the party within 15 days.
• Disqualification Independent Members: An independent member of a
House becomes disqualified to remain a member of the House if he joins
any political party after such election.
• Disqualification of Nominated Members: A nominated member of a House
becomes disqualified for being a member of the House if he joins any
political party after the expiry of six months from the date on which he
takes his seat in the House.
Exceptions
• The above disqualification on the ground of defection does not apply in the
following two cases:
• If a member goes out of his party as a result of a merger of the party with
another party. A merger takes place when two-thirds of the members of the
party have agreed to such merger.
• If a member, after being elected as the presiding officer of the House,
voluntarily gives up the membership of his party or rejoins it after he ceases
to hold that office.
Deciding Authority
• Any question regarding disqualification arising out of defection is to be
decided by the presiding officer of the House.
• Originally, the act provided that the decision of the presiding officer is final
and cannot be questioned in any court.
Kihoto Hollohan case (1983):
• In this case, Supreme Court declared this provision as unconstitutional on
the ground that it seeks to take away the jurisdiction of the Supreme Court
and the high courts.
• It held that the presiding officer, while deciding a question under the Tenth
Schedule, function as a tribunal.

40 | iLearn IAS
• Hence, presiding officer’s decision like that of any other tribunal, is subject
to judicial review on the grounds of mala fides, perversity, etc.
Rule-Making Power
• The presiding officer of a House is empowered to make rules to give effect
to the provisions of the Tenth Schedule.
• Further, he may direct that any willful contravention by any member of
such rules may be dealt with in the same manner as a breach of privilege
of the House.
• According to the rules made so, the presiding officer can take up a
defection case only when he receives a complaint from a member of the
House.
• Before taking the final decision, he must give the member (against whom
the complaint has been made) a chance to submit his explanation.
• He may also refer the matter to the committee of privileges for inquiry.
• Hence, defection has no immediate and automatic effect.
91st Amendment Act of 2003
• In the original act there was a provision for exemption from disqualification
in case of split by one-third members of legislature party.
• It was deleted by the 91st Amendment Act of 2003
• It means that the defectors have no protection on grounds of splits
PRELIMS QUESTION:
8.Consider the following statements regarding “Anti-defection law” in India:
1. Any question regarding disqualification arising out of defection is to be
decided by the Governor.
2. An independent member of a House becomes disqualified if he joins any
political party after election.
Which of the statements given above is/are correct?
(a) 1 only
(b) 2 only
(c) Both 1 and 2
(d) Neither 1 nor 2
Answer: B

Tuberculosis (TB)
Why in news?
• India registered a decline in the notification of tuberculosis (TB) cases by
about 85 per cent in April this year, after the imposition of the novel

41 | iLearn IAS
coronavirus disease (COVID-19) lockdown which led to a decline in access to
TB services in the country.
• India, has the highest TB burden across the globe, which accounts for 26 per
cent of the total global cases.
About TB
• Tuberculosis (TB) is caused by bacteria (Mycobacterium tuberculosis) that
most often affect the lungs.
Transmission:
• TB is spread from person to person through the air.
Risk of infection:
• People infected with TB bacteria have a 5–15% lifetime risk of falling ill with
TB.
• However, persons with compromised immune systems, such as people
living with HIV have a much higher risk of falling ill.
Drug resistance:
• Drug resistance emerges when anti-TB medicines are used inappropriately,
through incorrect prescription by health care providers, poor quality drugs,
and patients stopping treatment prematurely.
• Multidrug-resistant tuberculosis (MDR-TB) is a form of TB caused by
bacteria that do not respond to isoniazid and rifampicin, the 2 most
powerful, first-line anti-TB drugs.
• MDR-TB is treatable and curable by using second-line drugs.
Vaccination:
• Bacille Calmette-Guérin (BCG) is a vaccine for TB.
Add ons:
• ‘End TB Strategy’, adopted by the WHO is a blueprint for countries to end
the TB epidemic.
• Ending the TB epidemic by 2030 is among the health targets of the
Sustainable Development Goals.
• DOTS (Directly observed treatment, short course) is the name given to the
tuberculosis control strategy recommended by the World Health
Organization.
PRELIMS QUESTION:
9.Consider the following statements:
1. Tuberculosis (TB) is a viral disease that affect the lungs
2. It is spread from person to person through the air.
Which of the statements given above is/are correct?
(a) 1 only
42 | iLearn IAS
(b) 2 only
(c) Both 1 and 2
(d) Neither 1 nor 2
Answer: B

STARS project
Why in news?
• Union Cabinet has approved STARS project worth Rs 5718 crore.
About STARS:
• STARS is abbreviation for The Strengthening Teaching-Learning and Results
for States (STARS)).
• The project will get the financial support from the World Bank amounting
to US $ 500 million (approximately Rs. 3700 crore).
What is the focus of STARS project?
• The overall focus and components of the STARS project are aligned with the
objectives of the National Education Policy (NEP) 2020 of Quality Based
Learning Outcomes.
• The Project envisions improving the overall monitoring and measurement
activities in the Indian School Education System through interventions in
selected states.
• The project shifts focus from the provision of inputs and maintaining of
outputs to actual outcomes by linking the receipt and disbursement of
funds to these outcomes.
How will STARS project work?
• The STARS project seeks to support the states in developing, implementing,
evaluating, and improving interventions with direct linkages to improved
education outcomes and school to work transition strategies for improved
labour market outcomes.
• STARS project would be implemented as a new Centrally Sponsored Scheme
under the Department of School Education and Literacy, Ministry of
Education. (MOE).
• Under STARS, The Ministry of Education (MoE) will establish PARAKH, an
assessment centre that will set standards for the 60-odd examination
boards in the country as announced in the New Education Policy.
• The STARS project also aims to focus on initiatives of PM e-Vidya,
Foundational Literacy and Numeracy Mission and National Curricular and
Pedagogical Framework for Early Childhood Care and Education as part of
the Atmanirbhar Bharat Abhiyan.

43 | iLearn IAS
Where will STARS project be implemented?
• List of states where STARS project is implemented initially are Himachal
Pradesh, Rajasthan, Maharashtra, Madhya Pradesh, Kerala and Odisha.
• It has been envisaged that the STARS project will be implemented in five
more states in the near future namely Gujarat, Tamil Nadu, Uttarakhand,
Jharkhand and Assam.

PRELIMS QUESTION:
10.Consider the following statements regarding STARS project:
1. It is a world bank assisted Project in education sector.
2. It will be implemented as a centrally sponsored project.
Which of the statements given above is/are correct?
(a) 1 only
(b) 2 only
(c) Both 1 and 2
(d) Neither 1 nor 2
Answer: C

44 | iLearn IAS
Pulses Cultivation in India
Why in news?
• In a bid to cool down the recent hike in pulses prices, the Union
government plans to release 40,000 tonnes of tur dal from its buffer stock
into the retail market in small lots.
About pulses:
• Pulses are annual leguminous crops yielding between one and 12 grains or
seeds of variable size, shape and colour within a pod, used for both food
and feed.
• The term “pulses” is limited to crops harvested solely for dry grain,
thereby excluding crops harvested green for food, which are classified as
vegetable crops, as well as those crops used mainly for oil extraction and
leguminous crops that are used exclusively for sowing purposes.
• Besides serving as an important source of protein for a large portion of the
global population, pulses contribute to healthy soils and climate change
mitigation through their nitrogen-fixing properties.
• Pulses have low carbon and water footprints which make them an integral
part of the sustainable farming system.
Major Pulses in India:
• Bengal Gram (Desi Chick Pea / Desi Chana), Pigeon Peas (Arhar / Toor / Red
Gram), Green Beans (Moong Beans), Chick Peas (Kabuli Chana), Black
Matpe (Urad / Mah / Black Gram), Red Kidney Beans (Rajma), Black Eyed
Peas (Lobiya), Lentils (Masoor), White Peas (Matar) are major pulses grown
and consumed in India.
• India is the largest producer (25% of global production), consumer (27% of
world consumption) and importer (14%) of pulses in the world.
Production in India:
• Pulses account for around 20 per cent of the area under foodgrains and
contribute around 7-10 per cent of the total foodgrains production in the
country.
• Though pulses are grown in both Kharif and Rabi seasons, Rabi pulses
contribute more than 60 per cent of the total production.
• Gram is the most dominant pulse having a share of around 40 per cent in
the total production followed by Tur/Arhar at 15 to 20 per cent and
Urad/Black Matpe and Moong at around 8-10 per cent each.
• Madhya Pradesh, Maharashtra, Rajasthan, Uttar Pradesh and Karnataka are
the top five pulses producing States.

45 | iLearn IAS
Price Support:
• Government supports pulses production by providing Minimum Support
Prices (MSP) and procuring them from the farmers through National
Agricultural Cooperative Marketing Federation of India (NAFED) and more
recently through Small Farmers Agri Consortium (SFAC).
PRELIMS QUESTION:
11.Consider the following statements regarding Pulses in India:
1. India is self-sufficient in the production of pulses.
2. Procurement of pulses at minimum support price is done primarily through
Food Corporation of India.
3. Pulses have low carbon and water footprints during cultivation.
Which of the statements given above is/are correct?
(a) 1 and 2 only
(b) 2 and 3 only
(c) 3 only
(d) 1,2 and 3
Answer: C

Indian Rhinoceros
Why in news?
• Rhino Task Force of Assam and World Wildlife Fund India (WWF India) have
been undertaking steps to study pathogens found in fresh rhino dung
samples in Assam, Uttar Pradesh, and West Bengal.
About Indian rhinoceros:
• The Indian rhinoceros (Rhinoceros unicornis), also called the Indian rhino,
greater one-horned rhinoceros or great Indian rhinoceros, is a rhinoceros
species native to the Indian subcontinent.
• It is listed as Vulnerable on the IUCN Red List, as populations are
fragmented and restricted to less than 20,000 km2.
• The most important habitat of rhinos is the alluvial Terai-Duar savanna and
grasslands and riverine forest.
• As of 2019, a total of 2100-2200 mature individuals were estimated to live
in the wild.
• Indian rhinos once ranged throughout the entire stretch of the Indo-
Gangetic Plain, but excessive hunting and agricultural development reduced
its range drastically to 11 sites in northern India and southern Nepal.
• Kaziranga National Park with around 1800 individuals has the highest
number of individuals followed by Jaldapara National Park and Pobitora
Wildlife Sanctuary.

46 | iLearn IAS
Species characteristics:
• The largest sized rhinoceros reportedly range up to 4,000 kg and among
terrestrial land mammals native to Asia, Indian rhinos are second in size
only to the Asian elephant.
• They are also the second-largest living rhinoceros, behind only the white
rhinoceros.
• Indian rhinos have a thick grey-brown skin with pinkish skin folds and one
horn on their snout.
• The Indian rhino's single horn is present in both males and females and is
made of pure keratin.
PRELIMS QUESTION:
12.Consider the following statements regarding Indian Rhinoceros:
1. It is endemic to India.
2. They are largest species of rhinoceros.
Which of the statements given above is/are correct?
(a) 1 only
(b) 2 only
(c) Both 1 and 2
(d) Neither 1 nor 2
Answer: D

Gupkar Declaration
Why in news?
• National Conference chief Farooq Abdullah announced People's Alliance for
‘Gupkar Declaration’ after an all-party meet at his residence in Jammu and
Kashmir (J&K).
About Gupkar Declaration:
• Gupkar declaration was signed after an all-party meeting at the Gupkar
Road residence of NC supremo Farooq Abdullah.
• The resolution had parties unanimously agreeing to unify in their efforts to
protect and defend the identity, autonomy and special status of Jammu
and Kashmir.
• On 22 August 2020, leaders from the six political parties once again held a
meeting and signed the Gupkar II Declaration to renew their resolve to
adhere to the 2019 declaration that was signed.
• Under the Declaration parties committed to strive for the restoration of
Articles 370 and 35A, the Constitution of J-K and the restoration of the state
and cancellation of division of the state.

47 | iLearn IAS
PRELIMS QUESTION:
13.Gupkar declaration, recently in news, is related to the matters in the state/
union territory of:
(a) Puducherry
(b) Jammu and Kashmir
(c) Gujarat
(d) Nagaland
Answer: B

Clouded Leopard
Why in news?
• Meghalaya, Tripura and Mizoram — are taking steps to protect the
endangered clouded leopard.
About the Clouded Leopard:
• The clouded leopard occurs from the Himalayan foothills in Nepal, Bhutan
and India to Myanmar, southeastern Bangladesh, Thailand, Peninsular
Malaysia, to south of the Yangtze River in China.
• It is the state animal of the Indian state of Meghalaya.
Range:
• In India, it occurs in Sikkim, northern West Bengal, Meghalaya subtropical
forests, Tripura, Mizoram, Manipur, Assam, Nagaland and Arunachal
Pradesh.
• They have been spotted in Pakke Tiger Reserve (Arunachal),
Khangchendzonga Biosphere Reserve (Sikkim), Manas National Park
(Assam), Dampa Tiger Reserve (Mizoram), Eaglenest Wildlife Sanctuary
(Arunachal) and Singchung-Bugun Village Community Reserve (Arunachal),
Meghalaya's Nongkhyllem National Park and Balpakram-Baghmara
landscape (Meghalaya)
Threats and Conservation:
• IUCN status: Vulnerable
• Its total population is suspected to be fewer than 10,000 mature
individuals, with a decreasing population trend.
• They are threatened by habitat loss and poaching.
PRELIMS QUESTION:
14.Consider the following statements regarding Clouded Leopards:
1. They are endemic to India.
2. They are listed as vulnerable in the IUCN Red data list.
Which of the statements given above is/are correct?
(a) 1 only
48 | iLearn IAS
(b) 2 only
(c) Both 1 and 2
(d) Neither 1 nor 2
Answer: B

Real Estate Appellate Tribunal (REAT)


About REAT
• Real Estate (Regulation and Development) Act, 2016 mandates state
governments to form Real Estate Appellate Tribunal (REAT) to ensure faster
resolution of disputes.
• Government of two or more States or Union territories can also establish
one single Appellate Tribunal.
Purpose of establishment:
• Parties aggrieved by the Real Estate Regulatory Authority (RERA) order can
appeal before REAT and REAT has to adjudicate such cases within 60 days.
• Civil Courts have been prevented from exercising jurisdiction on such
matters.
Composition
• Tribunal shall consist of a Chairperson and not less than two whole time
Members of which one shall be a Judicial member and other shall be a
technical or administrative member.
• The chairperson is or has been a Judge of a High Court.
Appointment
• The Chairperson and members of the Appellate Tribunal shall be appointed
by the concerned state Government in consultation with the Chief Justice
of High Court.
Terms of office:
• Chairperson and members of the Appellate Tribunal shall hold office for a
term not exceeding 5 years; and shall not be eligible for re-appointment.
Powers:
• For the purpose of discharging its functions, the tribunal have the powers
of a civil court.
• Tribunal shall not be bound by the procedure laid down by the Code of Civil
Procedure and shall be guided by the principles of natural justice.
Appeal:
• If any of the parties is not satisfied with the REAT order, they can file an
appeal to the High Court within 60 days.

49 | iLearn IAS
PRELIMS QUESTION:
15.With reference to ‘Real Estate Appellate Tribunal (REAT)’, consider the
following statements:
1. The chairperson and members of the Tribunal are appointed by President of
India in consultation with Supreme Court
2. Appeals against the orders of the tribunal directly lies with Supreme Court
only
Which of the statements given above is/are correct?
(a) 1 only
(b) 2 only
(c) Both 1 and 2
(d) Neither 1 nor 2
Answer: D

National Health Authority


Why in news?
• Vipul Aggarwal, Deputy CEO of National Health Authority hails India's
COVID-19 response saying that "it stands out".
About NHA:
• It was created through an executive order of the cabinet, for better
implementation of Pradhan Mantri Jan Arogya Yojana (PM-JAY).
• It replaced "National Health Agency" as an attached office to Ministry of
Health & Family Welfare.
Structure of NHA
• The post of the NHA CEO will be upgraded to that of a Secretary to the
Government of India with full financial powers.
• Till now, all funds released by the National Health Agency, which is
implementing the PM-JAY, were done through the Ministry of Health.
• NHA’s governing board will be chaired by the Minister for Health and Family
Welfare, while its members will include NITI Aayog CEO and NHA CEO.
Functions of NHA
• NHA will be responsible for PM-JAY’s operational guidelines, fixing the
ceiling of premium amounts, and developing mechanisms for strategic
purchase of healthcare from the private sector.
• It is also tasked with building a health information technology platform, and
working with the Insurance Regulatory and Development Authority.
About PM-JAY (Ayushman Bharat)
• PM-JAY is being described as the world’s largest health insurance scheme.

50 | iLearn IAS
• It is a cashless and paperless access to health services up to Rs. 5 lakhs per
year to poor and vulnerable people identified as per the socio-economic
caste census.
PRELIMS QUESTION:
16.Consider the following statements regarding National health authority:
1. It will be responsible for implementation of Pradhan Mantri Jan Arogya
Yojana (PM-JAY).
2. It is a statutory body created through an act of parliament.
Which among the above statements is/are correct?
(a) 1 only
(b) 2 only
(c) Both 1 and 2
(d) Neither 1 nor 2
Answer: A

Asia-Pacific Economic Cooperation


Why in News?
• New Zealand will host Asia-Pacific Economic Cooperation (APEC) meeting in
2021.
About APEC
• Asia-Pacific Economic Cooperation (APEC) is an inter-governmental forum
for 21 Pacific Rim member economies that promotes free trade throughout
the Asia-Pacific region.
• APEC is headquartered in Singapore.
• APEC was established in 1989 in response to the growing interdependence
of Asia-Pacific economies and the advent of regional trade blocs in other
parts of the world; and to establish new markets for agricultural products
and raw materials beyond Europe.
What Does APEC Do?
• APEC ensures that goods, services, investment and people move easily
across borders.
• Members facilitate this trade
through faster customs
procedures at borders; more
favorable business climates
behind the border; and aligning
regulations and standards across
the region.

51 | iLearn IAS
How APEC arrives at decision making?
• In APEC, all economies have an equal say and decision-making is reached by
consensus. There are no binding commitments or treaty obligations.
Who are the members of APEC?
• APEC's 21 member economies are Australia; Brunei Darussalam; Canada;
Chile; Hong Kong, China; Indonesia; Japan; Republic of Korea; Malaysia;
Mexico; New Zealand; Papua New Guinea; Peru; The Philippines; The
Russian Federation; Singapore; Chinese Taipei; Thailand; United States of
America and Viet Nam.
• India is NOT a member of APEC.
PRELIMS QUESTION:
17.Consider the following statements regarding APEC:
1. India is NOT a member of APEC.
2. APEC is headquartered in Singapore.
Which among the above statements is/are correct?
(a) 1 only
(b) 2 only
(c) Both 1 and 2
(d) Neither 1 nor 2
Answer: C

Global Hunger Index 2020


Why in news?
• India ranks 94th out of 107 in Global Hunger Report 20s0.
About Global Hunger Index
• The report is prepared jointly published by Concern Worldwide, and
Welthungerhilfe (WHH).
• It termed the level of hunger in India "serious".
• The GHI score is calculated on four indicators –
o Undernourishment: Share of the population with insufficient caloric
intake.
o Child wasting: the share of children under the age of five who are
wasted (that is, who have low weight for their height, reflecting
acute under nutrition)
o Child stunting: children under the age of five who have low height
for their age, reflecting chronic under nutrition
o child mortality: the mortality rate of children under the age of five.

52 | iLearn IAS
• India fares worst in child wasting (low weight for height, reflecting acute
undernutrition) and child stunting (low height for age, reflecting chronic
undernutrition), which together make up a third of the total score.
• Although it is still in the poorest category, however, child stunting has
actually improved significantly, from 54% in 2000 to less than 35% now.
• Child wasting, on the other hand, has not improved in the last two decades,
and is rather worse than it was decade ago.
• India has improved in both child mortality rates, which are now at 3.7%,
and in terms of undernourishment, with about 14% of the total population
which gets an insufficient caloric intake.
• In the region of south, east and south-eastern Asia, the only countries
which fare worse than India are Timor-Leste, Afghanistan and North Korea.
• China has moved to a 'low' severity category and Sri Lanka is in the
'moderate' severity category.
PRELIMS QUESTION
18.Which among the following organization released Global Hunger Index 2019:
1. Concern Worldwide
2. Welthungerhilfe
3. Food and Agricultural Organization
Which of the statements given above is/are correct?
(a) 1 and 2 only
(b) 1 and 3
(c) 2 and 3
(d) 1,2 and 3
Answer: A

53 | iLearn IAS
Caspian Sea
Why in news?
• Russia began military exercises in the central waters of the Caspian Sea
north of the Azerbaijani capital Baku, insisting there was no threat to
neighbouring states as Armenia and Azerbaijan battle over the Nagorno-
Karabakh region.
About Caspian Sea
• The Caspian Sea is the world's largest inland body of water, variously
classed as the world's largest lake or a full-fledged sea.
• It is an endorheic basin (a basin without outflows) located between Europe
and Asia, to the east of the Caucasus Mountains and to the west of the
broad steppe of Central Asia.
• The sea has a surface area of 371,000 km2 (143,200 sq mi) (excluding the
detached lagoon of Garabogazköl) and a volume of 78,200 km3 (18,800 cu
mi). It has a salinity of approximately 1.2% (12 g/l), about a third of the
salinity of most seawater.
• It is bounded by Kazakhstan to the northeast, Russia to the northwest,
Azerbaijan to the west, Iran to the south, and Turkmenistan to the
southeast.
• The Caspian Sea is home to a wide range of species and may be best known
for its caviar and oil industries. Pollution from the oil industry and dams on
rivers draining into the Caspian Sea have had negative effects on the
organisms living in the sea. In the Caspian Basin, world’s first offshore wells
and machine-drilled wells were made in Bibi-Heybat Bay, near Baku,
Azerbaijan.

54 | iLearn IAS
PRELIMS QUESTION
19.Which of the countries border Caspian Sea:
1. Kazakhstan
2. Kyrgyzstan
3. Tajikistan
4. Turkmenistan
Select the correct answer using the code given below:
(a) 1 and 3 only
(b) 1 and 4 only
(c) 2 and 3 only
(d) 1,2,3 and 4
Answer: B

Thailand
Why in news?
• Thailand is witnessing large scale student protests against country’s
powerful monarchy and the country’s Thai Army-backed prime minister
Prayuth Chan-ocha.
• About Thailand:
• Thailand, formerly known as Siam, is located at the centre of the
Indochinese Peninsula.
• Thailand is the world's 50th-largest country by land area, and the 22nd-
most-populous country in the world.
• The capital and largest city is Bangkok.
• Thailand is bordered to the north by Myanmar and Laos, to the east by Laos
and Cambodia, to the south by the Gulf of Thailand and Malaysia, and to
the west by the Andaman Sea and the southern extremity of Myanmar.
• Nominally, Thailand is a constitutional monarchy and parliamentary
democracy; however, in recent history, its government has experienced
multiple coups and periods of military dictatorships.
• Thailand is a founding member of the Association of Southeast Asian
Nations (ASEAN) and remains a major ally of the United States.
PRELIMS QUESTION
1.Which of the countries share a land border with Thailand?
1. Myanmar
2. Cambodia
3. Laos
55 | iLearn IAS
4. Vietnam
Select the correct answer using the code given below:
(a) 1 and 3 only
(b) 2,3 and 4 only
(c) 1,2 and 3 only
(d) 1,2,3 and 4
Answer: C

Zoji La
Why in News?
• Blasting process launched for construction of Zojil Tunnel.
About Zoji La Pass
• Zoji La is a high mountain pass located in the Kargil district of Ladakh.
• The pass links Leh and Srinagar and provides an important link between
Union Territories of Ladakh and Kashmir.
• Zojila pass, at an altitude of nearly 3539 meters, remains closed during
winters due to heavy snowfall, cutting off Ladakh region from Kashmir.
About Zoji La tunnel
• In 2018, the Zojila tunnel project was launched.
• The project is strategically important because it will provide all weather
connectivity between between Srinagar, Kargil and Leh.
• The tunnel is Asia's longest and strategic bi-directional tunnel with a
length of 14.2 km.

56 | iLearn IAS
PRELIMS QUESTION
2.Zoji La tunnel, recently in news, connects:
(a) Kashmir with Ladakh
(b) Jammu with Kashmir
(c) Spit valley with Ladakh
(d) Dharasu with Nelong Valley.
Answer: A

Integrated Farming
About integrated farming:
• Integrated Farming, integrated production or Integrated Farm Management
is a whole farm management system which aims to deliver more
sustainable agriculture.
• It is a farming system where high-quality organic food, feed, fibre and
renewable energy are produced by using resources such as soil, water, air
and nature as well as regulating factors to farm sustainably and with as
little polluting inputs as possible.
• It is a dynamic approach which can be applied to any farming system
around the world.
• Integrated Farming combines the best of modern tools and technologies
with traditional practices according to a given site and situation.
PRELIMS QUESTION
3.Consider the following statements regarding integrated farming:
1. It does not use modern tools and technologies and focuses on improvement
of traditional practices.
2. It can be applied to any farming system around the world.
Which among the above statements is/are correct?
(a) 1 only
(b) 2 only
(c) Both 1 and 2
(d) Neither 1 nor 2
Answer: B
NOTE:
In the last compilation (October week 1), in the article "central Asia" it was given
that Tajikistan was a double land-locked country. However, Uzbekistan is the only
double land locked country in central Asia. The error is regretted.

57 | iLearn IAS

You might also like